Jump to content

IloveMaths's Content

There have been 162 items by IloveMaths (Search limited from 05-06-2020)



Sort by                Order  

#453788 [Topic]Hỏi đáp về việc Vẽ Hình!

Posted by IloveMaths on 29-09-2013 - 06:53 in Vẽ hình trên diễn đàn

thử phát   :luoi:  :luoi:  :luoi:  :luoi:

 

57643955.emyeujpd.jpg

 




#418043 PT-HPT-BPT Tuyển tập các bài toán sưu tầm từ Mathslink.ro

Posted by IloveMaths on 12-05-2013 - 19:43 in Phương trình - Hệ phương trình - Bất phương trình

Giả sử $x_{1}> x_{2}\Rightarrow x_{3}> x_{1}\Rightarrow x_{3}> x_{2}\Rightarrow x_{2}> x_{4}\Rightarrow x_{3}> x_{4}\Rightarrow x_{5}> x_{3}\Rightarrow x_{5}> x_{4}\Rightarrow x_{4}> x_{1}\Rightarrow x_{5}> x_{3}>x_{2}> x_{4}>x_{1}$

 

Chú ý: $x_{3}> x_{1}\Rightarrow x_{1}+x_{2}> x_{3}+x_{4}$  ( vô lí)

Lập luận tương tự $x_{1}< x_{2}$ vô lí

Vậy $x_{1}=x_{2}$

Do $x_{1}=x_{2} \Rightarrow x_{1}=x_{2}=x_{3}=x_{4}=x_{5}$

Do đó ta chỉ cần  giải phương trình:

$3^5x_{1}^5=3x_{1}\Rightarrow x_{1}=0$  hoặc $x_{1}=\frac{1}{3}$ hoặc $x_{1}=-\frac{1}{3}$

 

 

Giả sử $x_{1}> x_{2}\Rightarrow x_{3}> x_{1}\Rightarrow x_{3}> x_{2}\Rightarrow x_{2}> x_{4}\Rightarrow x_{3}> x_{4}\Rightarrow x_{5}> x_{3}\Rightarrow x_{5}> x_{4}\Rightarrow x_{4}> x_{1}\Rightarrow x_{5}> x_{3}>x_{2}> x_{4}>x_{1}$

 

Chú ý: $x_{3}> x_{1}\Rightarrow x_{1}+x_{2}> x_{3}+x_{4}$  ( vô lí)

Lập luận tương tự $x_{1}< x_{2}$ vô lí

Vậy $x_{1}=x_{2}$

Do $x_{1}=x_{2} \Rightarrow x_{1}=x_{2}=x_{3}=x_{4}=x_{5}$

Do đó ta chỉ cần  giải phương trình:

$3^5x_{1}^5=3x_{1}\Rightarrow x_{1}=0$  hoặc $x_{1}=\frac{1}{3}$ hoặc $x_{1}=-\frac{1}{3}$

 

Ai xem bai nay co dung ko




#423702 PT-HPT-BPT Tuyển tập các bài toán sưu tầm từ Mathslink.ro

Posted by IloveMaths on 04-06-2013 - 12:01 in Phương trình - Hệ phương trình - Bất phương trình



Bài toán 39: Tìm tất cả các nghiệm của PT $ (a^{2}+2b^{2}+2ab)(a^{2}+2b^{2}-2ab)=3b^{2}(2(a^{2}-b^{2})^{2}(a^{2}+b^{2}))^{\frac{1}{3}} $

 

 

Ta có:

$VP=(a^2+2b^2+2ab)(a^2+2b^2-2ab)=((a+b)^2+b^2)((a-b)^2+b^2)=((a^2-b^2)^2)+b^4+b^2.2.(a^2+b^2)\geq 3b^2.\sqrt[3]{2.(a^2-b^2)^2.(a^2+b^2))}=VT$

Dấu đẳng thức xảy ra khi 

$(a^2-b^2)^2=2b^2(a^2+b^2)=b^4$

-Nếu $b=0 \Rightarrow a=0$ thỏa mãn

-Nếu $b\neq 0\Rightarrow 2a^2+2b^2=b^2\Rightarrow 2a^2=-b^2$ (vô lí)

Vậy nghiêm của phương trình là:

(a;b)=(0;0)

:icon6:  :icon6:  :icon6:




#414258 PT-HPT-BPT Tuyển tập các bài toán sưu tầm từ Mathslink.ro

Posted by IloveMaths on 22-04-2013 - 14:12 in Phương trình - Hệ phương trình - Bất phương trình

 

 
Bài toán 15: Giải hệ PT $\left\{ \begin{array}{l}{x^3} - 3x - 2 = 2 - y\\{y^3} - 3y - 2 = 4 - 2z\\{z^3} - 3z - 2 = 6 - 3x\end{array} \right.$

 

 

Hệ tương đương:

$(x+1)^2(x-2)=2-y$

$(y+1)^2(y-2)=2(2-z)$

$(z+1)^2(z-2)=3(2-x)$

$\Rightarrow (z+1)^2.(x+1)^2.(y+1)^2.(x-2)(y-2)(z-2)=-6(x-2)(y-2)(z-2)$

- TH1:

$(x-2)(y-2)(z-2)=0$

- TH2:

$(x-2)(y-2)(z-2)\neq 0$ $\Rightarrow (x+1)^2.(y+1)^2.(z+1)^2=-6$  (  vô lí)




#415579 PT-HPT-BPT Tuyển tập các bài toán sưu tầm từ Mathslink.ro

Posted by IloveMaths on 30-04-2013 - 14:58 in Phương trình - Hệ phương trình - Bất phương trình

 

 
Bài toán 16: Giải hệ PT $\left\{ \begin{array}{l}{({x_3} + {x_4} + {x_5})^5} = 3{x_1}\\{({x_4} + {x_5} + {x_1})^5} = 3{x_2}\\{({x_5} + {x_1} + {x_2})^5} = 3{x_3}\\{({x_1} + {x_2} + {x_3})^5} = 3{x_4}\\{({x_2} + {x_3} + {x_4})^5} = 3{x_5}\end{array} \right.$

 

 

Giả sử $x_{1}> x_{2}\Rightarrow x_{3}> x_{1}\Rightarrow x_{3}> x_{2}\Rightarrow x_{2}> x_{4}\Rightarrow x_{3}> x_{4}\Rightarrow x_{5}> x_{3}\Rightarrow x_{5}> x_{4}\Rightarrow x_{4}> x_{1}\Rightarrow x_{5}> x_{3}>x_{2}> x_{4}>x_{1}$

 

Chú ý: $x_{3}> x_{1}\Rightarrow x_{1}+x_{2}> x_{3}+x_{4}$  ( vô lí)

Lập luận tương tự $x_{1}< x_{2}$ vô lí

Vậy $x_{1}=x_{2}$

Do $x_{1}=x_{2} \Rightarrow x_{1}=x_{2}=x_{3}=x_{4}=x_{5}$

Do đó ta chỉ cần  giải phương trình:

$3^5x_{1}^5=3x_{1}\Rightarrow x_{1}=0$  hoặc $x_{1}=\frac{1}{3}$ hoặc $x_{1}=-\frac{1}{3}$




#414271 Dãy số-Giới hạn Tuyển tập sưu tầm các bài toán từ Mathlinks.ro

Posted by IloveMaths on 22-04-2013 - 16:57 in Dãy số - Giới hạn

 

Bài toán 19: 

  1. Xét dãy các số sau : $49;4489;444889;...$ với quy luật là số đứng sau xác định bằng cách thêm $48$ vào vị trí giữa của số đứng trước đó. Chứng minh rằng đây là dãy chính phương

 

Ta chứng minh bài toán tổng quát hơn ,  

Chúng minh với mọi a thuộc $N$  thì:

$A=$$44...48...89$ ( trong đó có $a+1$ chữ số 4 và $a$ chữ số 8 )  là số chính phương 

Thật vây: 

Đặt $x=11....1$ ( $a+1$ chứ số 1)

Do đó $A=4x(9x+1)+8x+1=(6x+1)^2$




#415512 BĐT -Tuyển tập các bài toán sưu tầm từ Mathlinks.ro

Posted by IloveMaths on 30-04-2013 - 09:55 in Bất đẳng thức - Cực trị



 




 

Bài toán 14: Cho $a,b,c$ là độ dài 3 cạnh của 1 tam giác.Gọi $\beta$ là góc giữa cạnh $a$ và $c$.Chứng minh: (22)

\[\frac{{{b^2} + {c^2}}}{{{a^2}}} > \frac{{2\sqrt 3c \sin \beta  - a}}{{b + c}}\]

 

 

 

$\frac{{{b^2} + {c^2}}}{{{a^2}}} > \frac{{2\sqrt 3c \sin \beta - a}}{{b + c}}$

$\Leftrightarrow \frac{b^2+c^2}{a^2}+\frac{a}{b+c}>\frac{2.\sqrt{3}.sin\beta.c }{b+c} \Leftrightarrow (b^2+c^2)(b+c)+a^3>2.\sqrt{3}.c.sin\beta .a^2=2.\sqrt{3}.abc.sinA$

 

 

Ta có: $sinA\leq 1$  nên ta chỉ cần chứng minh: 

$(b^2+c^2)(b+c)+a^3>2.\sqrt{3}abc\Leftrightarrow a^3+b^3+c^3+bc(b+c)> 2.\sqrt{3}abc$

Theo bất đẳng thức Cauchy và bất đẳng thức trong tam giác:

$a^3+b^3+c^3+bc(b+c)>3abc+abc=4abc>2.\sqrt{3}.a.bc$$a^3+b^3+c^3+bc(b+c)>3abc+abc=4abc>2.\sqrt{3}.a.bc$

 




#415538 BĐT -Tuyển tập các bài toán sưu tầm từ Mathlinks.ro

Posted by IloveMaths on 30-04-2013 - 11:00 in Bất đẳng thức - Cực trị

Bài 16: Cho $a,b,c>0$ thỏa mãn $a+b+c \ge \dfrac{a}{b}+\dfrac{b}{c}+\dfrac{c}{a}$.Chứng minh rằng:

$\dfrac{a^3c}{b(a+c)} +\dfrac{b^3a}{c(a+b)}+\dfrac{c^3b}{a(b+c)} \ge \dfrac{3}{2}$

--

Bài 14 chưa ai giải,em đăng cho song song hai bài :P

Theo Bất đẳng thức Cauchy:

$\sum \frac{a^3c}{b(a+c)}+\sum \frac{a+c}{4}+\sum \frac{b}{2c}\geq \sum \frac{3a}{2}$

Do đó:

$\dfrac{\sum (a^3c)}{b(a+c)}\geq \sum \frac{3a}{2}- \frac{\sum a}{2}-\sum \frac{b}{2c} \geq \sum \frac{a}{2}+(\sum \frac{a}{2}-\sum \frac{b}{2c})\geq \sum \frac{a}{2}$

Từ giả thiết:

$a+b+c\geq \frac{a}{b}+\frac{b}{c}+\frac{c}{a}\geq 3$$\Rightarrow Q.E.D$




#407183 Phần mềm vẽ hình

Posted by IloveMaths on 23-03-2013 - 12:10 in Vẽ hình trên diễn đàn

http://upanh.com/vie...&id=4vlf8t1j3lr



#375489 : $\sum \frac{a^{2}-bc}{b+2c+d}...

Posted by IloveMaths on 06-12-2012 - 07:25 in Bất đẳng thức và cực trị

ồn ào quá,mọi người làm hay cãi vã đấy



#404464 Đề thi HSG 11 Đà Nẵng 2012-2013

Posted by IloveMaths on 12-03-2013 - 18:02 in Thi HSG cấp Tỉnh, Thành phố. Olympic 30-4. Đề thi và kiểm tra đội tuyển các cấp.

$x+y+25=8(\sqrt{x-1}+\sqrt{y-5})$
$\Leftrightarrow x-1+y-5+2\sqrt{(x-1)(y-5)}+31=8(\sqrt{x-1}+\sqrt{y-5})+2\sqrt{(x-1)(y-5)}
\Leftrightarrow (\sqrt{x-1}+\sqrt{y-5})^{2}+31=8(\sqrt{x-1}+\sqrt{y-5})+2\sqrt{(x-1)(y-5)}

\Leftrightarrow 2\sqrt{(x-1)(y-5)}=(\sqrt{x-1}+\sqrt{y-5}-4)^{2}+15\geqslant 15 \Leftrightarrow A\geq 7,5$



Thế dấu bằng xảy ra khi nào



#416266 Đề thi HSG TP.Hà Nội 2013-2014 (5-4-2013)

Posted by IloveMaths on 03-05-2013 - 21:49 in Tài liệu - Đề thi

Chỗ này là thế nào ạ

Đúng rồi mà em :

$MA_{1}+M'A_{1}> MM'=2$

.................................

$MA_{2013}+M'A_{2013}> MM'=2$

 

$\Rightarrow VT > 2013 .2$

 

 

Sorry, quên, MM' là đuong kính của đường tròn (O,1)




#416122 Đề thi HSG TP.Hà Nội 2013-2014 (5-4-2013)

Posted by IloveMaths on 02-05-2013 - 22:29 in Tài liệu - Đề thi

 

Bài 3:(2 điểm)

Cho a,b,c > 0 thỏa mãn:$\frac{1}{a}+\frac{2}{b}+\frac{3}{c}=3$.Chứng minh:

$\frac{27a^{2}}{c(c^{2}+9a^{2})}+\frac{b^{2}}{a(4a^{2}+b^{2})}+\frac{8c^{2}}{b(9b^{2}+4c^{2})}\geq \frac{3}{2}$

 

 

Đặt b= 2x , c= 3y

BĐT trở thành:$\frac{a^2}{y(y^2+a^2)}+\frac{x^2}{a(a^2+x^2)}+\frac{y^2}{x(x^2+y^2)}\geq \frac{3}{2}$ . Với $\frac{1}{x}+\frac{1}{y}+\frac{1}{a}=3$

 

Tiếp tục đặt $p=\frac{1}{a};\frac{1}{x}=q;r=\frac{1}{y}\Rightarrow p+q+r=3$

Do đó ta cần chứng minh:

$\frac{r^3}{p^2+r^2}+\frac{p^3}{p^2+q^2}+\frac{q^3}{q^2+r^2}\geq \frac{3}{2}$

Sử dụng bổ đề:

$\frac{r^3}{p^2+r^2}\geq r-\frac{p}{2}$

$\Rightarrow Q.E.D$




#416126 Đề thi HSG TP.Hà Nội 2013-2014 (5-4-2013)

Posted by IloveMaths on 02-05-2013 - 22:54 in Tài liệu - Đề thi

 

Bài 5:(1 điểm)

Cho 2013 điểm A1,A2,...,A2013 và đường tròn (O;1) tùy ý nằm trên mặt phẳng.Cmr trên (O) đó,ta luôn tìm được một điểm M sao cho MA1+MA2+...+MA2013$\geq$ 2013

 

 

Giả sử trên đương tròn (O;1) khong tìm đuoc điểm M nào thỏa mãn MA1+MA2+...+MA2013 $\geq 2013$

Lúc đó có điểm M và gọi M' là điểm đối xứng của M quá O  thỏa mãn

MA1+MA2+...+MA2013  < 2013

M'A1+M'A2+...+M'A2013 < 2013

 

 

Công theo vế được :

(MA1+M'A1)+...+(MA2013 +M'A2013 )  < 2013 . 2

Nhưng theo Bất đẳng thức tam giác thì MA1+M'A1 > MM'=2

Do đó VT > 2013 .2 

Vô lí 

Vậy luôn tồn tại điểm M thỏa mãn như đề bài




#457189 Đề thi chọn đội tuyển toán tỉnh Bà Rịa Vũng Tàu 2013-2014

Posted by IloveMaths on 12-10-2013 - 19:46 in Thi HSG cấp Tỉnh, Thành phố. Olympic 30-4. Đề thi và kiểm tra đội tuyển các cấp.



 

Ngày thi: 10/10/2013
Thời gian: 180 phút 
 
Câu 1. (4 điểm)
Giải phương trình $$8x^3-12x^2+5x=\sqrt[3]{3x-2}$$

 

$8x^3-12x^2+5x=\sqrt[3]{3x-2}\Leftrightarrow (2x-1)^3+(2x-1)=\sqrt[3]{3x-2}+(3x-2)$

:luoi:  :icon6:  Đến đây là Ok rùi 




#407742 chứng minh rằng $a^{2}+b^{2}+c^{2}+2abc+1...

Posted by IloveMaths on 25-03-2013 - 12:13 in Bất đẳng thức và cực trị

$a^{2}+b^{2}+c^{2}+\frac{9abc}{a+b+c}\geq 2(ab+bc+ca)$, theo cầu chì 3 số thì $(abc+aba+1)(a+b+c)\geq 9abc$, từ đây ta có đ.p.c.m  đẳng thức xảy ra khi và chỉ khgi a=b=c=1

BÀi làm sai rồi




#452222 Đề thi chọn Đội tuyển HSG tỉnh Nghệ An

Posted by IloveMaths on 22-09-2013 - 08:22 in Thi HSG cấp Tỉnh, Thành phố. Olympic 30-4. Đề thi và kiểm tra đội tuyển các cấp.

 

ĐỀ THI CHỌN ĐỘI TUYỂN DỰ THI HỌC SINH GIỎI LỚP 12 THPT (NGHỆ AN)

Ngày thứ nhất (11.10.2011)

Bài 3.(4,0 điểm)

Cho tam giác nhọn ABC có trực tâm H. Phân giác ngoài của góc BHC cắt các cạnh AB, AC lần lượt tại D, E. Gọi K là giao điểm của phân giác góc A của tam giác ABC và đường tròn ngoại tiếp tam giác ADE (K khác A). Chứng minh rằng hai tam giác BHK và CHK có diện tích bằng nhau.

 

:luoi:  :luoi: chém tí 

Dê dàng chứng minh AK là đường kính của đường tròn ngoại tiếp tam giác ADE 

Gọi $DK\cap BH=M;EK\cap CH=N;AC\cap BH=S;AB\cap CH=F$

De dàng chứng minh được tứ giác HMKN là hình bình hành

Do đó ta cần chúng minh :

$S_{\Delta BMK}=S_{\Delta NCK}\Rightarrow BM.MK=KN.NC\Leftrightarrow \frac{BM}{KN}=\frac{NC}{MK}\Leftrightarrow \frac{BD}{DF}=\frac{CE}{SE}\Leftrightarrow \frac{BH}{HF}=\frac{HC}{SH}\Leftrightarrow BH.SH=HC.HF$ 

$\Rightarrow Q.E.D$

:luoi:




#420765 Đề tuyển sinh THPT chuyên Lam Sơn (Sở GD và ĐT Thanh Hóa)

Posted by IloveMaths on 24-05-2013 - 20:29 in Tài liệu - Đề thi

dự đoán Max = 3 mình dùng tam thức bậc 2 hả ?  :icon1:

:icon6:  :icon6:  :icon6:  :icon6: 

Thử vài giá trị thôi .............. :lol: 




#420796 Đề tuyển sinh THPT chuyên Lam Sơn (Sở GD và ĐT Thanh Hóa)

Posted by IloveMaths on 24-05-2013 - 21:33 in Tài liệu - Đề thi

Câu 3: (2,0 điểm)

2. Tìm tất cả các số nguyên tố $p$ để $4p^{2}+1$ và $6p^{2}+1$ cũng là số nguyên tố.

 

Xét so dư của p khi chia cho 5 , ta có các so du là 0,1,2,3,4

- Nếu $p=5k+1(k\geq 1)\Rightarrow 4p^2+1=4.(5k+1)^2+1\vdots 5$ và $p>5\Rightarrow$ không là số nguyên tô

- Néu $p=5k+2(k\geq 1)\Rightarrow 6p^2+1=6(5k+2)^2+1\vdots 5$ và $p>5\Rightarrow$ không là sô nguyên tố

-Nếu $p=5k+3(k\geqslant 1)\Rightarrow 6p^2+1=6.(5k+3)^2+1\vdots 5$ và $p>5\Rightarrow$ không là số nguyên tô

-Nếu $p=5k+4\Rightarrow 4p^2+1=4.(5k+4)^2+1\vdots 5$ và $p>5\Rightarrow$ không là số nguyên tô

Vậy p = 5




#420718 Đề tuyển sinh THPT chuyên Lam Sơn (Sở GD và ĐT Thanh Hóa)

Posted by IloveMaths on 24-05-2013 - 17:30 in Tài liệu - Đề thi

 

Câu 2: (2,0 điểm) Cho phương trình: $ax^{2}+bx+c=0 (a\neq 0)$ có hai nghiệm $x_{1},x_{2}$ thỏa mãn điều kiện: $0\leq x_{1}\leq x_{2}\leq 2$. Tính GTLN của biểu thức:

                             $Q=\frac{2a^{2}-3ab+b^{2}}{2a^{2}-ab+ac}$

 

 

Theo Vi-ét:  $x_{1}+x_{2}=\frac{-b}{a};x_{1}x_{2}=\frac{c}{a}$

Ta có:

$Q=\frac{2a^{2}-3ab+b^{2}}{2a^{2}-ab+ac}$=$\frac{{2-\frac{3.b}{a}+(\frac{b}{a})^2}}{2-\frac{b}{a}+\frac{c}{a}}$=$\frac{2+3(x_{1}+x_{2})+(x_{1}+x_{2})^2}{2+(x_{1}+x_{2})+x_{1}.x_{2}}$

Dự đoán Max = 3

Do đó ta cần chứng minh:

$2+3(x_{1}+x_{2})+(x_{1}+x_{2})^2\leq 6+3(x_{1}+x_{2})+3.x_{1}.x_{2}\Leftrightarrow x_{1}^2+x_{2}^2-x_{1}.x_{2}\leq 4$

Xét hai truong hop:

-Nếu $x_{1}=0\Rightarrow x_{2}^2\leq 4$ ( đúng do $x_{2}\leq 2$ ) 

-Nếu $x_{1}\neq 0\Rightarrow x_{1}^2+x_{2}^2-x_{1}.x_{2}\leq x_{1}^2+x_{2}^2-x_{1}.x_{1}= x_{2}^2\leq 4$ ( đúng do $x_{2}\leq 2$ )

 

Vậy Max = 3 . Dấu bằng xảy ra khi $(x_{1};x_{2})=(0;2),(2;2)$

 

:luoi:  :luoi:  :luoi:  :icon6:  :icon6:  :icon6:  :icon6:  :icon6:  :icon6:  :icon6:  :icon6:  :icon6:  :icon6:  :icon6:  :icon6:  :icon6:  :icon6:  :icon6:  :icon6:  :icon6:  :icon6:  :icon6:  :icon6:  :icon6:




#432710 Cho $x+y=1$. Tìm GTLN của $\frac{x^3}{x+1...

Posted by IloveMaths on 04-07-2013 - 10:03 in Bất đẳng thức và cực trị

Cho $x,y$ là các số thực không âm thỏa mãn $x+y=1$. Tìm giá trị lớn nhất của biểu thức:

$$P=\frac{x^3}{x+1}+\frac{y^3}{y+1}$$

Áp dụng bất đẳng thức Cauchy:

$\frac{x^3}{x+1}+\frac{x+1}{4}+\frac{1}{2}+\frac{x}{2}\geq x$

$\frac{y^3}{y+1}+\frac{y+1}{4}+\frac{1}{2}+\frac{y}{2}\geq y$

$\Rightarrow \frac{x^3}{x+1}+\frac{y^3}{y+1}\geq \frac{3}{4}$




#403551 [MO2013] - trận 22 - PT, BPT, HPT, HBPT

Posted by IloveMaths on 10-03-2013 - 09:56 in Thi giải toán Marathon dành cho học sinh Chuyên Toán 2013

Dễ thấy (0;0;0) là một nghiệm của hệ phương trinh
do đó ta chỉ cần xét khi x,y,z khác 0
Ta có: $x+y+z=xyz\Rightarrow x+y=z(xy-1)$
Mặt khác: $\frac{156x}{x^2+1}=\frac{65y}{y^2+1}=\frac{60z}{z^2+1}$ nên
để hệ có nghiệm thì x,y,z phải cùng dấu.
Xét hai trường hợp:
TH1: $xy=1 \Rightarrow x+y=0$ ( vô lí do x,y cùng dấu)
TH2: $xy\neq 1\Rightarrow z=\frac{x+y}{xy-1}\Rightarrow z=\frac{x+y}{xy-1}$
Do đó: $\frac{156x}{x^2+1}=\frac{65y}{y^2+1}=\frac{60z}{z^2+1}$
$\Leftrightarrow \frac{780x}{5(x^2+1)}=\frac{780y}{12(y^2+1)}=\frac{z}{13(z^2+1)}$$\Leftrightarrow \frac{x}{5(x^2+1)}=\frac{y}{12(y^2+1)}=\frac{z}{13(z^2+1)}$
$\Leftrightarrow \frac{x}{5(x^2+1)}=\frac{y}{12(y^2+1)}=\frac{(x+y)(xy-1)}{13(x^2+1)(y^2+1)}$
Từ đó:
$13x(y^2+1)=5(x+y)(xy-1) ; 12x(y^2+1)=5y(x^2+1)$
Do vậy:
$\left\{\begin{matrix} 12x(y^2+1)=5y(x^2+1) & & \\ 13xy^2+13x)=5(x^2y-x+y^2x-y)& & \end{matrix}\right.$
$\Leftrightarrow \left\{\begin{matrix} 5y(x^2+1)=12x(y^2+1) & & \\ x(8y^2+18)=5y(x^2-1) & & \end{matrix}\right. \Rightarrow \frac{4y^2+9}{6(y^2+1)}=\frac{x^2-1}{x^2+1} \Rightarrow 4x^2y^2+4y^2+9x^2+9=6x^2y^2-6y^2+6x^2-6
\Rightarrow x^2(2y^2-3)-10y^2-15=0 $
lại có : $12x(y^2+1)=5y(x^2+1)\Rightarrow 144x^2(y^2+1)^2=25y^2(x^2+1)^2$
Đặt
$x^2=a(a>0);y^2=b(b>0)$
Do vậy ta có hệ sau:
$\left\{\begin{matrix} a(2b-3)=10b+15 & & \\ 144a(b+1)^2=25b(a+1)^2& & \end{matrix}\right.$
Nếu $b=\frac{3}{2}\Rightarrow 10b+15=0\Rightarrow b=\frac{-3}{2}$ ( vô lí) do vậy $b\neq \frac{3}{2}$
từ đó: $a=\frac{10b+15}{2b-3}$ $\Rightarrow 144.\frac{10b+15}{2b-3}.(b+1)^2=25b(\frac{10b+15}{2b-3}+1)^2=25b(\frac{12(b+1)}{2b-3})^2=25.144.b.(b+1)^2.\frac{1}{(2b-3)^2}\Rightarrow 10b+15=\frac{25b}{2b-3}\Rightarrow 20b^2-30b+30b-45=25b\Rightarrow 20b^2-25b-45=0\Rightarrow b=\frac{9}{4} ; b=-1$
do b>0 nen b=-1 loại , vậy b=$\frac{9}{4}\Rightarrow y=\frac{3}{2};y=\frac{-3}{2}$

Với $y=\frac{3}{2}\Rightarrow x=5;z=1$
Với $y=\frac{-3}{2}\Rightarrow x=-5;z=-1$

Vậy hệ phương trinh có :
(x,y,z)=(5;$\frac{3}{2}$;z=1)
(x,y,z)=(-5;$-\frac{3}{2}$;z=-1)
và là các hoán vị của 0

 

Hoán vị của 0 là thế nào?

 

S = 7+3*10 = 37
















 




#414025 Đề Thi Phát Hiện HSG Lớp 8 Năm Học: 2012-2013

Posted by IloveMaths on 20-04-2013 - 23:03 in Tài liệu - Đề thi

2a)

* Bổ đề:  $1^3+2^3+....+n^3=(1+2+3+...+n)^2$

Dễ chứng minh bằng quy nạp , Áp dụng là xong

2b)

Giả sử tồn tại tồn tại hình vuông nhu vây.Ta có:

Số tạo thành từ 2012 số 0 và 2013 chia hết cho 3 nhưng không chia hết cho 9 nên không thể là số chính phương.Vậy không tồn tại tam giác như vậy

Xử luôn câu 3 b)

$x^5-x^4y-13x^3y^2+13x^2y^3+36xy^4-36y^5=(x-y)(x^4-13x^2y^2+36y^4) =(x-y)(x^2-4y)(x^2-9y)=(x-y)(x-2y)(x-3y)(x+2y)(x+3y)$

 

Dễ thây 33 chỉ phân tích ra tích các số nguyên khác nhau nhiều nhât là 3 sô. Vậy phương trinh không có nghiêm nguyên 




#414019 Đề Thi Phát Hiện HSG Lớp 8 Năm Học: 2012-2013

Posted by IloveMaths on 20-04-2013 - 22:58 in Tài liệu - Đề thi

2a)

* Bổ đề:  $1^3+2^3+....+n^3=(1+2+3+...+n)^2$

Dễ chứng minh bằng quy nạp , Áp dụng là xong




#377551 $$\sqrt{\frac{b+c-a}{a}}+...

Posted by IloveMaths on 14-12-2012 - 18:28 in Bất đẳng thức - Cực trị

đặt a+b=x;b+c=y,c+a=z
$\Rightarrow \sqrt{\frac{b+c-a}{a}}+\sqrt{\frac{a+c-b}{b}}+\sqrt{\frac{a+b-c}{c}}\geqslant 3\Leftrightarrow \sqrt{\frac{y+z-x}{x+y-z}}+\sqrt{\frac{x+y-z}{x+z-y}}+\sqrt{\frac{x+z-y}{y+z-x}}\geqslant 3$
(dung theo AM-GM)